Классические теоремы дифференциального исчисления — различия между версиями

Материал из Викиконспекты
Перейти к: навигация, поиск
м (Доделал правило Лопиталя, но мне не нравится доказательство, пусть кто-нибудь еще над ним поработает.)
м (rollbackEdits.php mass rollback)
 
(не показано 6 промежуточных версий 6 участников)
Строка 3: Строка 3:
 
{{В разработке}}
 
{{В разработке}}
  
= Теорема Ферма о значении производной в экстремальной точке =
+
== Теорема Ферма о значении производной в экстремальной точке ==
 
{{Определение
 
{{Определение
 
|definition=
 
|definition=
Точка <tex> x_0 </tex> называется '''точкой локального минимума''', если <tex> \forall x \in \dot{O}(x_0) \ f(x) \ge f(x_0) </tex>.
+
Точки минимума и максимума:
<br />
+
* Точка <tex> x_0 </tex> называется '''точкой локального минимума''', если <tex> \forall x \in \dot{O}(x_0) \ f(x) \ge f(x_0) </tex>.
Точка <tex> x_0 </tex> называется '''точкой локального максимума''', если <tex> \forall x \in \dot{O}(x_0) \ f(x) \le f(x_0) </tex>.
+
* Точка <tex> x_0 </tex> называется '''точкой локального максимума''', если <tex> \forall x \in \dot{O}(x_0) \ f(x) \le f(x_0) </tex>.
 
}}
 
}}
  
Строка 17: Строка 17:
 
Ферма
 
Ферма
 
|statement=
 
|statement=
Пусть <tex> f(x) </tex> существует и дифференцируема в <tex> O(x_0) </tex>, и <tex> x_0 </tex> - точка локального экстремума. Тогда <tex> f'(x_0) = 0.</tex>
+
Пусть <tex> f(x) </tex> существует и дифференцируема в <tex> O(x_0) </tex>, и <tex> x_0 </tex> {{---}} точка локального экстремума. Тогда <tex> f'(x_0) = 0.</tex>
 
|proof=  
 
|proof=  
Рассмотрим случай, когда <tex> x_0 </tex> - точка локального минимума. Случай с локальным максимумом доказывается аналогично.
+
Рассмотрим случай, когда <tex> x_0 </tex> {{---}} точка локального минимума. Случай с локальным максимумом доказывается аналогично.
  
 
<tex dpi= "150"> \frac{\Delta y}{\Delta x} = \frac{f(x_0 + \Delta x) - f(x_0)}{\Delta x}</tex>; рассмотрим <tex> \Delta x \approx 0 </tex>.  
 
<tex dpi= "150"> \frac{\Delta y}{\Delta x} = \frac{f(x_0 + \Delta x) - f(x_0)}{\Delta x}</tex>; рассмотрим <tex> \Delta x \approx 0 </tex>.  
Строка 27: Строка 27:
 
Возможны 2 случая для <tex> \Delta x </tex>:
 
Возможны 2 случая для <tex> \Delta x </tex>:
  
1) <tex> \Delta x < 0 \Rightarrow \frac{\Delta y}{\Delta x} \le 0 \Rightarrow f'(x_0) \le 0 </tex>
+
#<tex> \Delta x < 0 \Rightarrow \frac{\Delta y}{\Delta x} \le 0 \Rightarrow f'(x_0) \le 0 </tex>
 +
#<tex> \Delta x > 0 \Rightarrow \frac{\Delta y}{\Delta x} \ge 0 \Rightarrow f'(x_0) \ge 0 </tex>
  
2) <tex> \Delta x > 0 \Rightarrow \frac{\Delta y}{\Delta x} \ge 0 \Rightarrow f'(x_0) \ge 0 </tex>
 
 
Отсюда, <tex> f'(x_0) = 0 </tex>.
 
Отсюда, <tex> f'(x_0) = 0 </tex>.
 
}}
 
}}
  
Замечание: обратная теорема не всегда верна, например, <tex> y(x) = x^3, y'(0) = 0,</tex> но <tex> y(0) </tex> - не экстремум.
+
Замечание: обратная теорема не всегда верна, например, <tex> y(x) = x^3, y'(0) = 0,</tex> но <tex> y(0) </tex> {{---}} не экстремум.
  
 
{{Определение
 
{{Определение
Строка 40: Строка 40:
 
}}
 
}}
  
= Теорема Ролля о нулях производной =
+
== Теорема Ролля о нулях производной ==
  
 
{{Теорема
 
{{Теорема
Строка 52: Строка 52:
 
Рассмотрим 2 случая:
 
Рассмотрим 2 случая:
  
1) Обе точки граничные, то есть <tex> x_1, x_2 </tex> находятся на концах отрезка. Тогда, так как <tex> f(a) = f(b) </tex>, то <tex> f_{max}[a; b] = f_{min}[a; b] </tex>. Значит, <tex> f(x) </tex> на <tex> [a; b] </tex> - константа, то есть <tex>\forall c \in (a; b) \ f'(c) = 0</tex>
+
1) Обе точки граничные, то есть <tex> x_1, x_2 </tex> находятся на концах отрезка. Тогда, так как <tex> f(a) = f(b) </tex>, то <tex> f_{max}[a; b] = f_{min}[a; b] </tex>. Значит, <tex> f(x) </tex> на <tex> [a; b] </tex> {{---}} константа, то есть <tex>\forall c \in (a; b) \ f'(c) = 0</tex>
  
 
2) Хотя бы одна из точек <tex> x_1, x_2 </tex> не граничная. Пусть это, например, <tex> x_1 </tex>. Тогда по теореме Ферма <tex> f'(x_1) = 0</tex>.
 
2) Хотя бы одна из точек <tex> x_1, x_2 </tex> не граничная. Пусть это, например, <tex> x_1 </tex>. Тогда по теореме Ферма <tex> f'(x_1) = 0</tex>.
Строка 59: Строка 59:
 
Замечание: для непрерывной функции на заданном отрезке ей принимаются все значения между двумя граничными значениями. Такое же свойство выполняется и для ее производной, хотя она может быть уже разрывной.
 
Замечание: для непрерывной функции на заданном отрезке ей принимаются все значения между двумя граничными значениями. Такое же свойство выполняется и для ее производной, хотя она может быть уже разрывной.
  
= Теорема Дарбу о промежуточных значениях производной =
+
== Теорема Дарбу о промежуточных значениях производной ==
  
 
{{Теорема
 
{{Теорема
Строка 73: Строка 73:
 
<tex> D \in [A; B] \Rightarrow g'(x_1) < 0, g'(x_2) > 0 </tex>.
 
<tex> D \in [A; B] \Rightarrow g'(x_1) < 0, g'(x_2) > 0 </tex>.
  
По определению производной, <tex dpi = '150'> g(x_1) = \frac{g(x_1 + \Delta x) - g(x_1)}{\Delta x} </tex>
+
По определению производной, <tex dpi = '150'> g'(x_1) = \frac{g(x_1 + \Delta x) - g(x_1)}{\Delta x} </tex>
  
 
При <tex> \Delta x \approx 0, \Delta x > 0 \ g(x_1 + \Delta x) < g(x_1) </tex>  
 
При <tex> \Delta x \approx 0, \Delta x > 0 \ g(x_1 + \Delta x) < g(x_1) </tex>  
Строка 79: Строка 79:
 
Аналогично рассмотрим <tex> g'(x_2) </tex>: при <tex> \Delta x \approx 0, \Delta x < 0 \ g(x_2 + \Delta x) < g(x_2) </tex>  
 
Аналогично рассмотрим <tex> g'(x_2) </tex>: при <tex> \Delta x \approx 0, \Delta x < 0 \ g(x_2 + \Delta x) < g(x_2) </tex>  
  
Функция <tex> g(x) </tex> - дифференцируема, а значит, также и непрерывна на <tex> [x_1, x_2] </tex>, поэтому на этом отрезке существуют минимальное и максимальное значения функции. Из двух предыдущих неравенств следует, что минимальное значение достигается не в граничной точке.
+
Функция <tex> g(x) </tex> {{---}} дифференцируема, а значит, также и непрерывна на <tex> [x_1, x_2] </tex>, поэтому на этом отрезке существуют минимальное и максимальное значения функции. Из двух предыдущих неравенств следует, что минимальное значение достигается не в граничной точке.
 
Пусть оно достигается в точке <tex> d \in (x_1; x_2) </tex>, тогда по теореме Ферма в этой точке <tex> g'(d) = 0</tex>. Значит, <tex> f'(d) = g'(d) + D = D </tex>.
 
Пусть оно достигается в точке <tex> d \in (x_1; x_2) </tex>, тогда по теореме Ферма в этой точке <tex> g'(d) = 0</tex>. Значит, <tex> f'(d) = g'(d) + D = D </tex>.
 
}}
 
}}
  
= Формула конечных приращений Лагранжа =
+
== Формула конечных приращений Лагранжа ==
  
 
{{Теорема
 
{{Теорема
 +
|id=lagrange
 
|author=
 
|author=
 
Лагранж
 
Лагранж
Строка 98: Строка 99:
 
}}
 
}}
  
= Формула конечных приращений Коши =
+
== Формула конечных приращений Коши ==
  
 
{{Теорема
 
{{Теорема
Строка 106: Строка 107:
 
Пусть <tex> f, g </tex> непрерывны на <tex> [a; b] </tex> и дифференцируемы на <tex> (a; b) </tex>, <tex> g'(x) \ne 0\ \forall x \in (a; b)</tex>. Тогда <tex> \exists c \in (a; b): </tex> <tex dpi = '150'> \frac{f(b) - f(a)}{g(b) - g(a)} = \frac{f'(c)}{g'(c)} </tex>.
 
Пусть <tex> f, g </tex> непрерывны на <tex> [a; b] </tex> и дифференцируемы на <tex> (a; b) </tex>, <tex> g'(x) \ne 0\ \forall x \in (a; b)</tex>. Тогда <tex> \exists c \in (a; b): </tex> <tex dpi = '150'> \frac{f(b) - f(a)}{g(b) - g(a)} = \frac{f'(c)}{g'(c)} </tex>.
 
|proof=
 
|proof=
Для начала, докажем, что дробь в левой части равенства определена: по теореме Лагранжа, <tex> g(b) - g(a) = g'(d)(b - a) </tex> для некоторого d, по условию, правая часть не равна нулю, значит, <tex>g(b) - g(a) \ne 0</tex>.
+
Для начала, докажем, что дробь в левой части равенства определена: по теореме Лагранжа, <tex> g(b) - g(a) = g'(d)(b - a) </tex> для некоторого <tex>d</tex>, по условию, правая часть не равна нулю, значит, <tex>g(b) - g(a) \ne 0</tex>.
  
 
Рассмотрим вспомогательную функцию <tex> F(x) = f(x) - f(a) - k(g(x) - g(a)), k = </tex> <tex dpi = '150'>\frac{f(b) - f(a)}{g(b) - g(a)} </tex>.
 
Рассмотрим вспомогательную функцию <tex> F(x) = f(x) - f(a) - k(g(x) - g(a)), k = </tex> <tex dpi = '150'>\frac{f(b) - f(a)}{g(b) - g(a)} </tex>.
Строка 119: Строка 120:
 
}}
 
}}
  
Замечание: при g(x) = x получаем частный случай формулы Коши - формулу Лагранжа.
+
Замечание: при <tex>g(x) = x</tex> получаем частный случай формулы Коши {{---}} формулу Лагранжа.
  
= Правило Лопиталя раскрытия неопределенностей =
+
== Правило Лопиталя раскрытия неопределенностей ==
  
 
Из формулы Коши можно получить раскрытие неопределенностей вида <tex> \frac{0}{0} </tex>, <tex> \frac{\infty}{\infty} </tex>(в числителе и знаменателе дроби получаются нулевые или бесконечные значения). Это правило называют '''правилом Лопиталя''':
 
Из формулы Коши можно получить раскрытие неопределенностей вида <tex> \frac{0}{0} </tex>, <tex> \frac{\infty}{\infty} </tex>(в числителе и знаменателе дроби получаются нулевые или бесконечные значения). Это правило называют '''правилом Лопиталя''':
Строка 137: Строка 138:
 
Подставляя туда <tex> f(a), g(a) </tex>, получаем требуемое равенство.
 
Подставляя туда <tex> f(a), g(a) </tex>, получаем требуемое равенство.
  
Случай с неопределенностью вида <tex> \frac{\infty}{\infty} </tex> доказывается аналогично.
+
Случай с неопределенностью вида <tex> \frac{\infty}{\infty} </tex> <s>доказывается аналогично.</s>
 +
Докажем теорему для неопределённостей вида <math>\left(\frac{\infty}{\infty}\right)</math>.
 +
 
 +
Пусть, для начала, предел отношения производных конечен и равен <math>A</math>. Тогда, при стремлении <math>x</math> к <math>a</math> справа, это отношение можно записать как <math>A+\alpha</math>, где <math>\alpha</math> — [[O-большое и o-малое|O]](1). Запишем это условие:
 +
: <math>\forall\varepsilon_{1}>0\, \exists \delta_{1}>0 : \forall x(0\le x-a<\delta_{1}\Rightarrow \left| \alpha(x)\right| <\varepsilon_{1})</math>.
 +
 
 +
Зафиксируем <math>t</math> из отрезка <math>[a,\;a+\delta_1]</math> и применим [[теорема Коши о среднем значении|теорему Коши]] ко всем <math>x</math> из отрезка <math>[a,\;t]</math>:
 +
: <math>\forall x\in [a;t]\ \exists c\in [a;\;x]\!:\frac{f(x)-f(t)}{g(x)-g(t)}=\frac{f'(c)}{g'(c)}</math>, что можно привести к следующему виду:
 +
: <math>\frac{f(x)}{g(x)}=\frac{1-\frac{g(t)}{g(x)}}{1-\frac{f(t)}{f(x)}}\cdot\frac{f'(c)}{g'(c)}</math>.
 +
 
 +
Для <math>x</math>, достаточно близких к <math>a</math>, выражение имеет смысл; предел первого множителя правой части равен единице (так как <math>f(t)</math> и <math>g(t)</math> — константы, а <math>f(x)</math> и <math>g(x)</math> стремятся к бесконечности). Значит, этот множитель равен <math>1+\beta</math>, где <math>\beta</math> — бесконечно малая функция при стремлении <math>x</math> к <math>a</math> справа. Выпишем определение этого факта, используя то же значение <math>\varepsilon</math>, что и в определении для <math>\alpha</math>:
 +
: <math>\forall \varepsilon_{1}>0\, \exists \delta_{2}>0\ : \forall x(0\le x-a<\delta_{2}\Rightarrow \left| \beta(x) \right| <\varepsilon_{1})</math>.
 +
 
 +
Получили, что отношение функций представимо в виде <math>(1+\beta)(A+\alpha)</math>, и <math>\left|\frac{f(x)}{g(x)}-A\right|<|A|\varepsilon_{1}+\varepsilon_{1}+\varepsilon_{1}^{2}</math>. По любому данному <math>\varepsilon</math> можно найти такое <math>\varepsilon_{1}</math>, чтобы модуль разности отношения функций и <math>A</math> был меньше <math>\varepsilon</math>, значит, предел отношения функций действительно равен <math>A</math>.
 +
 
 +
Если же предел <math>A</math> бесконечен (допустим, он равен плюс бесконечности), то
 +
: <math>\forall M>0\, \exists \delta_{1}>0 : \forall x(0\le x-a<\delta_{1}\Rightarrow\frac{f'(x)}{g'(x)}>2M)</math>.
 +
 
 +
В определении <math>\beta</math> будем брать <math>\varepsilon_{1} < \frac{1}{2}</math>; первый множитель правой части будет больше 1/2 при <math>x</math>, достаточно близких к <math>a</math>, а тогда <math>\frac{f(x)}{g(x)}>\frac{1}{2}\cdot 2M=M\Rightarrow \lim_{x\to a+}{\frac{f(x)}{g(x)}}=+\infty</math>.
 +
 
 +
 
 
}}
 
}}

Текущая версия на 19:07, 4 сентября 2022


Эта статья находится в разработке!

Теорема Ферма о значении производной в экстремальной точке

Определение:
Точки минимума и максимума:
  • Точка [math] x_0 [/math] называется точкой локального минимума, если [math] \forall x \in \dot{O}(x_0) \ f(x) \ge f(x_0) [/math].
  • Точка [math] x_0 [/math] называется точкой локального максимума, если [math] \forall x \in \dot{O}(x_0) \ f(x) \le f(x_0) [/math].


Сами значения [math] f(x_o) [/math] называются соответственно локальным минимумом и локальным максимумом.

Теорема (Ферма):
Пусть [math] f(x) [/math] существует и дифференцируема в [math] O(x_0) [/math], и [math] x_0 [/math] — точка локального экстремума. Тогда [math] f'(x_0) = 0.[/math]
Доказательство:
[math]\triangleright[/math]

Рассмотрим случай, когда [math] x_0 [/math] — точка локального минимума. Случай с локальным максимумом доказывается аналогично.

[math] \frac{\Delta y}{\Delta x} = \frac{f(x_0 + \Delta x) - f(x_0)}{\Delta x}[/math]; рассмотрим [math] \Delta x \approx 0 [/math].

Заметим, что, по определению локального минимума, [math] f(x_0 + \Delta x) - f(x_0) \ge 0 [/math].

Возможны 2 случая для [math] \Delta x [/math]:

  1. [math] \Delta x \lt 0 \Rightarrow \frac{\Delta y}{\Delta x} \le 0 \Rightarrow f'(x_0) \le 0 [/math]
  2. [math] \Delta x \gt 0 \Rightarrow \frac{\Delta y}{\Delta x} \ge 0 \Rightarrow f'(x_0) \ge 0 [/math]
Отсюда, [math] f'(x_0) = 0 [/math].
[math]\triangleleft[/math]

Замечание: обратная теорема не всегда верна, например, [math] y(x) = x^3, y'(0) = 0,[/math] но [math] y(0) [/math] — не экстремум.


Определение:
Корень уравнения [math]f'(x) = 0[/math] называется стационарной точкой.


Теорема Ролля о нулях производной

Теорема (Ролль):
Пусть [math] f(x) [/math] непрерывна на [math] [a; b] [/math], дифференцируема на [math](a, b)[/math] и [math]f(a) = f(b)[/math]. Тогда существует точка [math] c \in (a; b)[/math], такая, что [math] f'(c) = 0[/math].
Доказательство:
[math]\triangleright[/math]

[math] f(x) [/math] непрерывна на [math] [a; b] [/math], значит, у нее на этом отрезке существуют минимум и максимум. Пусть [math] x_1 [/math] — точка минимума, [math] x_2 [/math] — точка максимума.

Рассмотрим 2 случая:

1) Обе точки граничные, то есть [math] x_1, x_2 [/math] находятся на концах отрезка. Тогда, так как [math] f(a) = f(b) [/math], то [math] f_{max}[a; b] = f_{min}[a; b] [/math]. Значит, [math] f(x) [/math] на [math] [a; b] [/math] — константа, то есть [math]\forall c \in (a; b) \ f'(c) = 0[/math]

2) Хотя бы одна из точек [math] x_1, x_2 [/math] не граничная. Пусть это, например, [math] x_1 [/math]. Тогда по теореме Ферма [math] f'(x_1) = 0[/math].
[math]\triangleleft[/math]

Замечание: для непрерывной функции на заданном отрезке ей принимаются все значения между двумя граничными значениями. Такое же свойство выполняется и для ее производной, хотя она может быть уже разрывной.

Теорема Дарбу о промежуточных значениях производной

Теорема (Дарбу):
Пусть [math] f(x) [/math] дифференцируема на [math] [x_1; x_2], A = f'(x_1), B = f'(x_2)[/math]. Тогда [math] \forall D \in [A; B] \ \exists d \in [x_1; x_2]: D = f'(d) [/math]
Доказательство:
[math]\triangleright[/math]

Для определенности считаем, что [math] A \lt B [/math], обратный случай доказывается аналогично.

Рассмотрим вспомогательную функцию [math] g(x) = f(x) - Dx; g'(x) = f'(x) - D [/math]

[math] D \in [A; B] \Rightarrow g'(x_1) \lt 0, g'(x_2) \gt 0 [/math].

По определению производной, [math] g'(x_1) = \frac{g(x_1 + \Delta x) - g(x_1)}{\Delta x} [/math]

При [math] \Delta x \approx 0, \Delta x \gt 0 \ g(x_1 + \Delta x) \lt g(x_1) [/math]

Аналогично рассмотрим [math] g'(x_2) [/math]: при [math] \Delta x \approx 0, \Delta x \lt 0 \ g(x_2 + \Delta x) \lt g(x_2) [/math]

Функция [math] g(x) [/math] — дифференцируема, а значит, также и непрерывна на [math] [x_1, x_2] [/math], поэтому на этом отрезке существуют минимальное и максимальное значения функции. Из двух предыдущих неравенств следует, что минимальное значение достигается не в граничной точке.

Пусть оно достигается в точке [math] d \in (x_1; x_2) [/math], тогда по теореме Ферма в этой точке [math] g'(d) = 0[/math]. Значит, [math] f'(d) = g'(d) + D = D [/math].
[math]\triangleleft[/math]

Формула конечных приращений Лагранжа

Теорема (Лагранж):
Пусть [math] f [/math] непрерывна на [math] [a; b] [/math] и дифференцируема на [math] (a; b) [/math]. Тогда [math] \exists c \in (a; b): [/math] [math] \frac{f(b) - f(a)}{b - a} [/math] [math] = f'(c) [/math]
Доказательство:
[math]\triangleright[/math]

Рассмотрим вспомогательную функцию [math] g(x) = (f(x) - f(a)) - k(x - a), k = [/math] [math] \frac{f(b) - f(a)}{b - a}[/math].

Заметим, что [math] g(a) = g(b) = 0 [/math], значит, по теореме Ролля, [math] \exists c \in (a; b): g'(c) = 0 [/math].

Но [math] g'(x) = f'(x) - k [/math], значит, [math] f'(c) = k = [/math] [math]\frac{f(b) - f(a)}{b - a} [/math]
[math]\triangleleft[/math]

Формула конечных приращений Коши

Теорема (Коши):
Пусть [math] f, g [/math] непрерывны на [math] [a; b] [/math] и дифференцируемы на [math] (a; b) [/math], [math] g'(x) \ne 0\ \forall x \in (a; b)[/math]. Тогда [math] \exists c \in (a; b): [/math] [math] \frac{f(b) - f(a)}{g(b) - g(a)} = \frac{f'(c)}{g'(c)} [/math].
Доказательство:
[math]\triangleright[/math]

Для начала, докажем, что дробь в левой части равенства определена: по теореме Лагранжа, [math] g(b) - g(a) = g'(d)(b - a) [/math] для некоторого [math]d[/math], по условию, правая часть не равна нулю, значит, [math]g(b) - g(a) \ne 0[/math].

Рассмотрим вспомогательную функцию [math] F(x) = f(x) - f(a) - k(g(x) - g(a)), k = [/math] [math]\frac{f(b) - f(a)}{g(b) - g(a)} [/math].

[math] F(a) = F(b) = 0 [/math], значит, по теореме Ролля, [math] \exists c \in (a; b): F'(c) = 0 [/math].

Но [math] F'(x) = f'(x) - kg'(x) [/math], значит

[math] f'(c) = kg'(c) [/math]

[math] \frac{f'(c)}{g'(c)} = k = \frac{f(b) - f(a)}{g(b) - g(a)} [/math]
[math]\triangleleft[/math]

Замечание: при [math]g(x) = x[/math] получаем частный случай формулы Коши — формулу Лагранжа.

Правило Лопиталя раскрытия неопределенностей

Из формулы Коши можно получить раскрытие неопределенностей вида [math] \frac{0}{0} [/math], [math] \frac{\infty}{\infty} [/math](в числителе и знаменателе дроби получаются нулевые или бесконечные значения). Это правило называют правилом Лопиталя:

Теорема (правило Лопиталя):
Если при [math]x \rightarrow a[/math] [math]\frac{f(x)}{g(x)} = \frac{0}{0} [/math], то [math] \lim\limits_{x \rightarrow a} \frac{f(x)}{g(x)} = \lim\limits_{x \rightarrow a} \frac{f'(x)}{g'(x)} [/math]
Доказательство:
[math]\triangleright[/math]

Доопределим по непрерывности значения функций в точке [math] a [/math]: [math] f(a) = g(a) = 0 [/math].

По формуле Коши для малого отрезка [math] [a; x] [/math] выполняется равенство [math] \frac{f(x) - f(a)}{g(x) - g(a)} = \frac{f'(x)}{g'(x)} [/math].

Подставляя туда [math] f(a), g(a) [/math], получаем требуемое равенство.

Случай с неопределенностью вида [math] \frac{\infty}{\infty} [/math] доказывается аналогично. Докажем теорему для неопределённостей вида [math]\left(\frac{\infty}{\infty}\right)[/math].

Пусть, для начала, предел отношения производных конечен и равен [math]A[/math]. Тогда, при стремлении [math]x[/math] к [math]a[/math] справа, это отношение можно записать как [math]A+\alpha[/math], где [math]\alpha[/math] — O(1). Запишем это условие:

[math]\forall\varepsilon_{1}\gt 0\, \exists \delta_{1}\gt 0 : \forall x(0\le x-a\lt \delta_{1}\Rightarrow \left| \alpha(x)\right| \lt \varepsilon_{1})[/math].

Зафиксируем [math]t[/math] из отрезка [math][a,\;a+\delta_1][/math] и применим теорему Коши ко всем [math]x[/math] из отрезка [math][a,\;t][/math]:

[math]\forall x\in [a;t]\ \exists c\in [a;\;x]\!:\frac{f(x)-f(t)}{g(x)-g(t)}=\frac{f'(c)}{g'(c)}[/math], что можно привести к следующему виду:
[math]\frac{f(x)}{g(x)}=\frac{1-\frac{g(t)}{g(x)}}{1-\frac{f(t)}{f(x)}}\cdot\frac{f'(c)}{g'(c)}[/math].

Для [math]x[/math], достаточно близких к [math]a[/math], выражение имеет смысл; предел первого множителя правой части равен единице (так как [math]f(t)[/math] и [math]g(t)[/math] — константы, а [math]f(x)[/math] и [math]g(x)[/math] стремятся к бесконечности). Значит, этот множитель равен [math]1+\beta[/math], где [math]\beta[/math] — бесконечно малая функция при стремлении [math]x[/math] к [math]a[/math] справа. Выпишем определение этого факта, используя то же значение [math]\varepsilon[/math], что и в определении для [math]\alpha[/math]:

[math]\forall \varepsilon_{1}\gt 0\, \exists \delta_{2}\gt 0\ : \forall x(0\le x-a\lt \delta_{2}\Rightarrow \left| \beta(x) \right| \lt \varepsilon_{1})[/math].

Получили, что отношение функций представимо в виде [math](1+\beta)(A+\alpha)[/math], и [math]\left|\frac{f(x)}{g(x)}-A\right|\lt |A|\varepsilon_{1}+\varepsilon_{1}+\varepsilon_{1}^{2}[/math]. По любому данному [math]\varepsilon[/math] можно найти такое [math]\varepsilon_{1}[/math], чтобы модуль разности отношения функций и [math]A[/math] был меньше [math]\varepsilon[/math], значит, предел отношения функций действительно равен [math]A[/math].

Если же предел [math]A[/math] бесконечен (допустим, он равен плюс бесконечности), то

[math]\forall M\gt 0\, \exists \delta_{1}\gt 0 : \forall x(0\le x-a\lt \delta_{1}\Rightarrow\frac{f'(x)}{g'(x)}\gt 2M)[/math].
В определении [math]\beta[/math] будем брать [math]\varepsilon_{1} \lt \frac{1}{2}[/math]; первый множитель правой части будет больше 1/2 при [math]x[/math], достаточно близких к [math]a[/math], а тогда [math]\frac{f(x)}{g(x)}\gt \frac{1}{2}\cdot 2M=M\Rightarrow \lim_{x\to a+}{\frac{f(x)}{g(x)}}=+\infty[/math].
[math]\triangleleft[/math]